illegal drugs

This topic has expert replies
Senior | Next Rank: 100 Posts
Posts: 39
Joined: Wed Jan 20, 2010 12:29 pm
Thanked: 1 times
Followed by:1 members

illegal drugs

by sallespadua » Thu Feb 11, 2010 7:22 am
Doubt in this question:

The program to control the entry of illegal drugs into the country was a failure in 1987. If the program had been successful, the wholesale price of most illegal drugs would not have dropped substantially in 1987.

The argument in the passage depends on which of the following assumptions?

(A) The supply of illegal drugs dropped substantially in 1987.
(B) The price paid for most illegal drugs by the average consumer did not drop substantially in 1987.
(C) Domestic production of illegal drugs increased at a higher rate than did the entry of such drugs into the country.
(D) The wholesale price of a few illegal drugs increased substantially in 1987.
(E) A drop in demand for most illegal drugs in 1987 was not the sole cause of the drop in their wholesale price.

QA E

GMAT Instructor
Posts: 1578
Joined: Thu May 28, 2009 8:02 am
Thanked: 128 times
Followed by:34 members
GMAT Score:760

by Osirus@VeritasPrep » Thu Feb 11, 2010 7:25 am
I would choose E

A- If the supply dropped without a subsequent drop in demand, the prices would have risen

B- The price that drug users paid for drugs is irrelevant to the wholesale price of the drugs

C- This goes beyond the scope of the argument

D- The stimulus doesn't support this

E- Correct, this answer eliminates the possibility that a drop in demand accounted for the drop in prices for drugs.
https://www.beatthegmat.com/the-retake-o ... 51414.html

Brandon Dorsey
GMAT Instructor
Veritas Prep

Buy any Veritas Prep book(s) and receive access to 5 Practice Cats for free! Learn More.

User avatar
Legendary Member
Posts: 2109
Joined: Sun Apr 19, 2009 10:25 pm
Location: New Jersey
Thanked: 109 times
Followed by:79 members
GMAT Score:640

by money9111 » Thu Feb 11, 2010 8:23 am
E as well..and i didn't even read all of the statements
My goal is to make MBA applicants take onus over their process.

My story from Pre-MBA to Cornell MBA - New Post in Pre-MBA blog

Me featured on Poets & Quants

Free Book for MBA Applicants


User avatar
Legendary Member
Posts: 1275
Joined: Thu Sep 21, 2006 11:13 pm
Location: Arabian Sea
Thanked: 125 times
Followed by:2 members

by ajith » Thu Feb 11, 2010 9:36 am
sallespadua wrote:Doubt in this question:

The program to control the entry of illegal drugs into the country was a failure in 1987. If the program had been successful, the wholesale price of most illegal drugs would not have dropped substantially in 1987.

The argument in the passage depends on which of the following assumptions?

(A) The supply of illegal drugs dropped substantially in 1987.
(B) The price paid for most illegal drugs by the average consumer did not drop substantially in 1987.
(C) Domestic production of illegal drugs increased at a higher rate than did the entry of such drugs into the country.
(D) The wholesale price of a few illegal drugs increased substantially in 1987.
(E) A drop in demand for most illegal drugs in 1987 was not the sole cause of the drop in their wholesale price.
Conclusion: The program to control the entry of illegal drugs into the country was a failure
Assumption: The drop in price of illegal drugs shows the control is not working ( or availability of illegal goods increased)
The drop in price is not due to drop in demand

A - Not an assumption
B - Not an assumption
C - No such assumption is taken
D - Not very relevant
E - Straight forward, the only strong answer
Always borrow money from a pessimist, he doesn't expect to be paid back.

Newbie | Next Rank: 10 Posts
Posts: 1
Joined: Fri Jan 06, 2017 7:29 pm

test

by Srikapardhi » Thu Nov 09, 2017 7:08 pm
I read almost 10 answers for the same Question but yours is the best of all that helped me crack the logic. thank you much.
ajith wrote:
sallespadua wrote:Doubt in this question:

The program to control the entry of illegal drugs into the country was a failure in 1987. If the program had been successful, the wholesale price of most illegal drugs would not have dropped substantially in 1987.

The argument in the passage depends on which of the following assumptions?

(A) The supply of illegal drugs dropped substantially in 1987.
(B) The price paid for most illegal drugs by the average consumer did not drop substantially in 1987.
(C) Domestic production of illegal drugs increased at a higher rate than did the entry of such drugs into the country.
(D) The wholesale price of a few illegal drugs increased substantially in 1987.
(E) A drop in demand for most illegal drugs in 1987 was not the sole cause of the drop in their wholesale price.
Conclusion: The program to control the entry of illegal drugs into the country was a failure
Assumption: The drop in price of illegal drugs shows the control is not working ( or availability of illegal goods increased)
The drop in price is not due to drop in demand

A - Not an assumption
B - Not an assumption
C - No such assumption is taken
D - Not very relevant
E - Straight forward, the only strong answer

GMAT/MBA Expert

User avatar
GMAT Instructor
Posts: 7223
Joined: Sat Apr 25, 2015 10:56 am
Location: Los Angeles, CA
Thanked: 43 times
Followed by:29 members

by Scott@TargetTestPrep » Mon Nov 20, 2017 10:55 am
sallespadua wrote:Doubt in this question:

The program to control the entry of illegal drugs into the country was a failure in 1987. If the program had been successful, the wholesale price of most illegal drugs would not have dropped substantially in 1987.

The argument in the passage depends on which of the following assumptions?

(A) The supply of illegal drugs dropped substantially in 1987.
(B) The price paid for most illegal drugs by the average consumer did not drop substantially in 1987.
(C) Domestic production of illegal drugs increased at a higher rate than did the entry of such drugs into the country.
(D) The wholesale price of a few illegal drugs increased substantially in 1987.
(E) A drop in demand for most illegal drugs in 1987 was not the sole cause of the drop in their wholesale price.
While, generally, answering Critical Reasoning questions does not require the use of any sophisticated knowledge, understanding the logic of some Critical Reasoning questions requires a basic understanding of the economic laws of supply and demand. This question is such a question.

For anyone who is not familiar with the laws of supply and demand, here is a basic summary:

If the supply of a good increases and all other factors are held equal, normally the price of the good decreases. Conversely, if the supply of a good decreases, normally the price increases.

Demand affects price in the opposite way. If the demand for a good increases and all other factors are held equal, normally the price of the good increases. If demand decreases, normally the price decreases.

Now, let's analyze the argument in this question.

The argument mentions a "program to control the entry of illegal drugs into the country." This program is related to the laws of supply and demand in that the purpose of the program is to reduce the supply of illegal drugs in the country by controlling the entry of the drugs.

As discussed above, a reduction in supply should result in an increase in price, but the argument indicates that "the wholesale prices of most illegal drugs... dropped substantially." In other words, even though there was a program meant to reduce supply, the prices fell.

Since one would expect that a reduction in supply would result in price increases rather than price decreases, the author of the argument reasons that since the prices decreased, supply must not have decreased, and therefore, the argument concludes, the program must have failed.

Now, let's go to the answer choices to see which the author assumed in coming to that conclusion.

(A) The supply of illegal drugs dropped substantially in 1987.

Since the conclusion of the argument is that the program to reduce supply failed, this answer choice is not an assumption required for coming to that conclusion but rather is a statement in opposition to the conclusion.

(B) The price paid for most illegal drugs by the average consumer did not drop substantially in 1987.

This answer choice is incorrect in two ways.

The first is that the author is not concerned with the prices consumers pay, i.e., the retail prices of illegal drugs. The author is concerned with wholesale prices of illegal drugs and has already made the case that supply did not drop by pointing to the drop in wholesale prices.

The second is that the conclusion states that the program to reduce supply failed. Any drop in prices of illegal drugs would be consistent with that conclusion. Therefore, clearly, the argument does not rely on the assumption that the retail prices of illegal drugs did not drop.

(C) Domestic production of illegal drugs increased at a higher rate than did the entry of such drugs into the country.

This answer choice could be tempting because an increase in domestic production of illegal drugs would result in an increase in supply, even if there were a program in place to reduce supply from foreign sources by controlling entry of drugs into the country.

So, this answer choice could explain why, even though there was a program to reduce supply, the prices of illegal drugs decreased; supply from domestic sources could have offset any reduction in supply achieved by the program to limit entry.

However, we are not looking for an explanation. We are looking for an assumption. So, this choice is not our answer.

(D) The wholesale price of a few illegal drugs increased substantially in 1987.

The conclusion of the argument is that a program to reduce supply failed. Since a reduction in supply would push prices upward, price increases would indicate that the program to reduce supply had not failed but instead had succeeded. So, the conclusion does not depend on assuming that any prices of illegal drugs increased.

(E) A drop in demand for most illegal drugs in 1987 was not the sole cause of the drop in their wholesale price.

Correct. The argument's conclusion, that the program to reduce supply failed, is based on the fact that the prices of illegal drugs dropped. In arriving at its conclusion, the argument relies on the assumption that there was not some other reason for the decreases in the prices of illegal drugs. Therefore, that a drop in demand was not the sole cause of the price drop is an assumption upon which the argument depends.

The correct answer is E

Scott Woodbury-Stewart
Founder and CEO
[email protected]

Image

See why Target Test Prep is rated 5 out of 5 stars on BEAT the GMAT. Read our reviews

ImageImage